LSAT and Law School Admissions Forum

Get expert LSAT preparation and law school admissions advice from PowerScore Test Preparation.

 DlarehAtsok
  • Posts: 50
  • Joined: Nov 18, 2015
|
#39402
I get why C) is the right question, but I cannot think of any good reasons to disprove A). The sentence can be seen as an explanation for the observation that follows it (teachers fear computers ...).
 Eric Ockert
PowerScore Staff
  • PowerScore Staff
  • Posts: 164
  • Joined: Sep 28, 2011
|
#39634
The wording of answer choice (A) indicates the use of a causal argument. Loosely translated it reads: "It is presented as a possible cause for an observed effect that follows it."

For that to be the correct answer, the author would need to conclude that this is what's causing teachers' fear (or administrators' enthusiasm). But the author never provides any support for why teachers fear computers for this reason. The author merely states it as a given fact that this is the reason why teachers fear computers.

The argument then goes on to say that teachers and administrators are mistaken in thinking that computers will reduce the need for teachers. So this is the therefore the position that the argument is designed to refute.

Hope that helps!
 adkirk88
  • Posts: 9
  • Joined: Aug 08, 2019
|
#67609
Could you explain why C is more favorable than B? I think I know why but I want to be sure I'm thinking the right way...

Is it because the potential problem of reducing the amount of teachers is not solved entirely (the argument acknowledges that a reduction in teachers would require a reduction in courses) but rather the line of reasoning referred to in the stimulus is properly refuted? That computers can't increase the amount of courses offered while decreasing the amount of teachers since they are necessary for computerized instruction?
 James Finch
PowerScore Staff
  • PowerScore Staff
  • Posts: 943
  • Joined: Sep 06, 2017
|
#67903
Hi AD Kirk,

My reading of (B) is that it is talking about something that would be a problem for everyone, not just certain people, while the statement in question is set up as objectionable to teachers but as helpful to administrators. Moreover, there's no solution to it, it's simply a prediction that the argument is designed to show is untrue or unlikely to be true. So it's not objectively a "problem" to be "solved," but rather a premise that the argument is seeking to disprove. (C) clearly expresses this, making it correct.

Hope this helps!

Get the most out of your LSAT Prep Plus subscription.

Analyze and track your performance with our Testing and Analytics Package.